A formulation of continuity for bilinear forms

  • Thread starter Thread starter quasar987
  • Start date Start date
  • Tags Tags
    Continuity Forms
Click For Summary
The discussion revolves around the continuity of a bilinear form defined on a real Hilbert space, specifically addressing whether the condition of coercivity implies continuity. Condition (ii) is highlighted as a statement about continuity, but there is confusion regarding its equivalence to the general definition of continuity. The argument presented shows that if a sequence converges to zero in the product topology, then both components must converge to zero, thereby establishing continuity. The conversation concludes that while condition (ii) implies continuity, the reverse implication holds true under the context of boundedness in the unit ball. Overall, the relationship between coercivity and continuity in bilinear forms is clarified.
quasar987
Science Advisor
Homework Helper
Gold Member
Messages
4,796
Reaction score
32
[SOLVED] A formulation of continuity for bilinear forms

Homework Statement


My HW assignment read "Let H be a real Hilbert space and a: H x H-->R be a coninuous coersive bilinear form (i.e.
(i) a is linear in both arguments
(ii) There exists M>0 such that |a(x,y)|<M||x|| ||y||
(iii) there exists B such tthat a(x,x)>a||x||^2"

So apparently, condition (ii) is the statement about continuity. But I fail to see how this statement is equivalent to "a is continuous".

I see how (ii) here implies continuous, but not the opposite.

The Attempt at a Solution



Let z_n = (x_n,y_n)-->0. Then x_n-->0 and y_n-->0. So |a(x,y)|<M||x|| ||y|| implies a(x,y)-->0. a is thus continuous at 0, so it is so everywhere, being linear.
 
Last edited:
Physics news on Phys.org
I'm not quite sure what the question is. It seems to have been omitted. But just looking at your argument, z_n=(x_n,y_n)->0 doesn't imply x_n->0 or y_n->0. Does it?
 
Well, I'm making use of the fact that if (M,d) is metric space, then the product topology on M x M is generated by the metric

D((x1,y1),(x2,y2))=[d(x1,x2)² + d(y1,y2)²]^½

I conclude that the norm on H x H is

||(x,y)|| = [||x||² + ||y||²]^½

And now if (x_n,y_n)-->0, this means that

[||x_n||² + ||y_n||²]^½ --> 0,

which can only happen if x_n-->0 and y_n-->0.

------------

You're right, I have not actually typed the question entirely. It is because I was confused by the fact that they seem to imply that condition (ii) is equivalent to continuity. While (ii) implies continuity, does continuity implies (ii)?
 
Last edited:
Well, there is the theorem that a linear functional on a Hilbert space is continuous if and only if it's bounded...
 
You mean bounded in the unit ball?

In this case, you're right, it works. Because ||(x,y)|| = [||x||² + ||y||²]^½ <1 ==> ||x||, ||y||<1 and (ii) implies|a(x,y)|<M for all (x,y) in H x H such that ||(x,y)||<1.
 
Question: A clock's minute hand has length 4 and its hour hand has length 3. What is the distance between the tips at the moment when it is increasing most rapidly?(Putnam Exam Question) Answer: Making assumption that both the hands moves at constant angular velocities, the answer is ## \sqrt{7} .## But don't you think this assumption is somewhat doubtful and wrong?

Similar threads

Replies
5
Views
2K
  • · Replies 23 ·
Replies
23
Views
2K
  • · Replies 3 ·
Replies
3
Views
2K
  • · Replies 7 ·
Replies
7
Views
2K
  • · Replies 2 ·
Replies
2
Views
2K
  • · Replies 18 ·
Replies
18
Views
3K
  • · Replies 5 ·
Replies
5
Views
2K
  • · Replies 10 ·
Replies
10
Views
3K
  • · Replies 21 ·
Replies
21
Views
2K
  • · Replies 14 ·
Replies
14
Views
2K